Difference between revisions of "2017 AMC 12A Problems/Problem 15"

Line 8: Line 8:
 
\qquad\textbf{(D)}\ (3, 4)
 
\qquad\textbf{(D)}\ (3, 4)
 
\qquad\textbf{(E)}\ (4,5) </math>
 
\qquad\textbf{(E)}\ (4,5) </math>
 +
 +
==Solution==
 +
SOMNBODY DO THIS ONE RREE
  
 
==See Also==
 
==See Also==
 
{{AMC12 box|year=2017|ab=A|num-b=14|num-a=16}}
 
{{AMC12 box|year=2017|ab=A|num-b=14|num-a=16}}
 
{{MAA Notice}}
 
{{MAA Notice}}

Revision as of 19:14, 8 February 2017

Problem

Let $f(x) = \sin{x} + 2\cos{x} + 3\tan{x}$, using radian measure for the variable $x$. In what interval does the smallest positive value of $x$ for which $f(x) = 0$ lie?

$\textbf{(A)}\ (0,1)  \qquad \textbf{(B)}\ (1, 2) \qquad\textbf{(C)}\ (2, 3) \qquad\textbf{(D)}\ (3, 4) \qquad\textbf{(E)}\ (4,5)$

Solution

SOMNBODY DO THIS ONE RREE

See Also

2017 AMC 12A (ProblemsAnswer KeyResources)
Preceded by
Problem 14
Followed by
Problem 16
1 2 3 4 5 6 7 8 9 10 11 12 13 14 15 16 17 18 19 20 21 22 23 24 25
All AMC 12 Problems and Solutions

The problems on this page are copyrighted by the Mathematical Association of America's American Mathematics Competitions. AMC logo.png